Question

In: Economics

1. If total cost is given by TC = 500+ 10Q − 10Q2 + 0.5Q3 ,...

1. If total cost is given by TC = 500+ 10Q − 10Q2 + 0.5Q3 , then average variable cost is minimized at __________ units of output.

a. 20 b. 10 c. 50 d. 5 e. 100

12. If output is produced according to Q = K0.5 + 3L0.5, then this production process exhibits:

a. increasing returns to scale b. constant returns to scale c. decreasing returns to scale d. first decreasing and then increasing returns to scale e. first increasing and then decreasing returns to scale

13. Company XYZ has zero fixed costs and its average variable cost of cases is given by AVC = 5Q + 600, the marginal cost at an output level of 15 units is:

a. $625 b. $500 c. $750 d. $600 e. $550

14. Given a cost function C(Q) = 400 + 18Q + 6Q2 , what is the marginal cost function?

a. 18 + 12Q. b. 18 + 12Q2. c. 400 + 6Q2. d. 18Q + 6Q2

Solutions

Expert Solution

Answer:

Question1]

Total cost(TC) = 500 + 10Q -10Q2 + 0.5Q3

Average cost is found by dividing the total cost by Q units

Average cost(AC) = TC / Q = 10 - 10Q + 0.5Q2

Taking the first derivative of the average cost function

d(AC)/dQ = -10 + Q

Taking the second derivative of the above equation

d2(AC)/dq2 = 1

Hence the above function satisfies the conditions of minimum

Equating the first derivative to zero

-10 + Q = 0

Q = 10 units

Hence the average cost is minimized at 10 units

B] 10 units is correct answer.

Question2]

If output is produced according to Q = K0.5 + 3L0.5, then this production process exhibits:

C] decreasing returns to scale

Explanation:

Q = K0.5 + 3L0.5

On doubling the inputs:

Q' = 20.5K0.5 + 3 x 20.5 x L0.5

Q' = 20.5 x Q

Output less than doubles. Therefore, there exist decreasing returns to scale.

Question3]

AVC = 5Q + 600

TVC = AVC * Q = (5Q + 600) * Q = 5Q2 + 600Q

MC = dTVC/dQ = d(5Q2 + 600Q)/dQ = 10Q + 600

Calculate the marginal cost when output is 15 units -

MC = 10Q + 600

MC = (10*15) + 600 = 750

The marginal cost at an output level of 15 units is $750.

Hence, the correct answer is the option (c)$750.

Question4]

Marginal Cost is the change in total cost and a change in function found by differentiation
MC=dTC/dQ=18+12Q
MC=18+12Q

Correct option A] 18 + 12Q


Related Solutions

1. The following equation describes a firm’s total cost. TC = 500 + 10Q + Q2...
1. The following equation describes a firm’s total cost. TC = 500 + 10Q + Q2 a. If the firm is a price taker and other firms in the industry sell output at a price of $100, what price should the manager of this firm put on the product? b. What level of output should be produced to maximize profits (or minimize losses)? c. Should the firm keep producing or shut down? Hint: Is P ≥ AVC? d. Calculate profit...
The short-term total cost curve is as follows. TC=10+10Q^2
The short-term total cost curve is as follows. TC=10+10Q^2Lead short-term average cost curve (AC), average variable cost curve (AVC), and average fixed cost curve (AFC) into formulas, and draw pictures.
A monopolist has total cost TC = Q2 + 10Q + 100 and marginalcost MC...
A monopolist has total cost TC = Q2 + 10Q + 100 and marginal cost MC = 2Q + 10. It faces demand Q = 130 - P (so its marginal revenue is MR = 130 - 2Q). Its profit-maximizing price is$50$75$100
•1) The Short-Run Cost function of a company is given by the equation: TC = 500+80Q2+...
•1) The Short-Run Cost function of a company is given by the equation: TC = 500+80Q2+ 3Q3
 where:
 TC is the total cost and Q is the total quantity of output, both measured in millions of dollars.
 If the company produced 5 units of goods, its average fixed cost is equal? 2)The Short-Run Cost function of a company is given by the equation: TC = 500+80Q2+ 3Q3
 where:
 TC is the total cost and Q is the total quantity of...
2. The market for air conditioners has: Total Cost: TC = 20 + 10Q +(3/4)Q2 Marginal...
2. The market for air conditioners has: Total Cost: TC = 20 + 10Q +(3/4)Q2 Marginal Cost: MC = 10 + (3/2)Q Marginal Revenue: MR = 1,010 – 0.5Q Demand: Q = 4,040 – 4P 2a. If a monopoly controls the market, calculate the equilibrium price and quantity of air conditioners. 2b. Calculate the monopoly profits from part a. 2c. If the government imposed a tax of $80 per air conditioner that the monopoly sells, calculate the equilibrium new price...
Consider a firm with a short run Total Cost (TC) given by TC=2000 + 1000Q -...
Consider a firm with a short run Total Cost (TC) given by TC=2000 + 1000Q - 40Q2 + Q3 . What is the firms marginal cost? What is firm's shut down price?
If a representative firm with the total cost given by TC = 20 + 20q +...
If a representative firm with the total cost given by TC = 20 + 20q + 5q2 operates in a competitive industry where the short-run market demand and supply curves are given by ??=1,400−40? and ??=−400+20?, the number of firms operating in the short run will be: a. 140 b. 200 c. 280 d. 100
Suppose a perfectly competitive firm's short-run total cost (TC) is given by         TC = 200...
Suppose a perfectly competitive firm's short-run total cost (TC) is given by         TC = 200 + 4Q + 2Q2 where Q = output and 200 = fixed cost. As a result, MC = 4 + 4Q. Suppose the price of the firm's price is $24. a. How much should the firm produce in the short run to maximize its profits? b. How large will the firm's short-run profits be? Remember Profit = TR – TC.      c. Should the...
For a monopolist’s output: Demand: P= 90 – 4Q    Cost: TC = 10Q. For the profit...
For a monopolist’s output: Demand: P= 90 – 4Q    Cost: TC = 10Q. For the profit maximizing monopolist that charges the same price to all buyers: Compute P and Q. Compute the Lerner Index (Price Cost Markup). Use the Lerner Index formula. Compute price elasticity of demand at the price in answer (a). Explain the relationship between your answers to b and c. Compute the deadweight loss from monopoly? Compute the deadweight loss from monopoly that Gordon Tullock would calculate?...
Suppose that the cost function of some manufacturer is TC(q) = 160 + 8q + 10q^2...
Suppose that the cost function of some manufacturer is TC(q) = 160 + 8q + 10q^2 . Find expressions for the firm’s ATC, AVC, AFC, and MC curves. Sketch the ATC, AVC, and MC curves. At what output level does the firm’s ATC reach its minimum point? What can you say about the marginal product curve (for the variable factor; e.g.,MPL) that must underlie this cost function? Briefly explain.
ADVERTISEMENT
ADVERTISEMENT
ADVERTISEMENT